LSAT and Law School Admissions Forum

Get expert LSAT preparation and law school admissions advice from PowerScore Test Preparation.

User avatar
 Dave Killoran
PowerScore Staff
  • PowerScore Staff
  • Posts: 5852
  • Joined: Mar 25, 2011
|
#44050
Setup and Rule Diagram Explanation

This is a Grouping: Defined-Moving, Unbalanced: Overloaded game.

This is a fairly standard Grouping game, except for the fact that there is a chairperson, and the chairperson affects the size of each group. Thus, the game actually has two different possible groupings:

J94_Game_#3_#setup_diagram 1.png
The vertical bar has been placed between the tenant and homeowner groups to make the group separation in each scenario more clear.

The last four rules are each conditional, and each is relatively easy to diagram:

  • Third rule: ..... F :arrow: Q

    Fourth rule: ..... G :arrow: K

    Fifth rule: ..... J :dbl: M

    Sixth rule: ..... M :dblline: P
The combination of the fourth and fifth rules leads to two interesting inferences. Because selecting G forces K to be selected, the JM block cannot be selected with G as that would mean that four tenants would be selected for the committee, a violation of the rules. Thus, G cannot be selected with either J or M:
  • ..... ..... G :dblline: J

    ..... ..... G :dblline: M

The combination of the fifth and sixth rules allows for another inference:

  • ..... ..... J :dblline: P
Thus, if J and M are selected, P cannot be selected (leaving only Q, R, and S left as homeowners possibly on the committee). Therefore, if either J or M is the chairperson, the five committee members must be J and M as the two tenants, and Q, R, and S as the three homeowners. This inference is tested on questions #14 and #17.

The combination of the information above leads to the final setup for the game:

J94_Game_#3_#setup_diagram 1.png
Rules:
  • F :arrow: Q

    G :arrow: K

    J :dbl: M

    M :dblline: P
Inferences:
  • G :dblline: J

    G :dblline: M

    J :dblline: P

    JC
    or :arrow: J, M, Q, R, S
    MC

Finally, this game contains eight questions, a unique number in modern LSAT Games history. All other games contain either five, six, or seven questions. The presence of eight questions meant that this game represented one-third of all the questions in this section, and thus a test taker hoping for a reasonable score in the Games section had to do this game.
You do not have the required permissions to view the files attached to this post.
 BBischoff
  • Posts: 1
  • Joined: Dec 21, 2017
|
#42518
Hi Powerscore Team,

I'm having some difficulty with my diagram for this game. I was able to correctly answer the first six problems but it took me far far too long and then I was left befuddled by the final two (I probably could have brute force worked them out but since my diagram was so clearly missing some key facet I figured it was probably better just to get to the root of the issue). This is what I came up with to start:

2 or more of FGJKM
2 or more of PQRS

F :arrow: Q
G :arrow: K
J :dbl: M
M :dblline: P

I then drew the following inferences of
Not K then not G
Not Q then not F
J :dblline: P
if J or M :arrow: not G because G would pull in K

I tried to set up my diagram as an "in group" of 5 and an "out group" of 4

_ _ _ _ _ | _ _ _ _

The diagram helped a lot with say question 15 where I knew that GJM couldn't be in the ingroup now which left just K, but it seemed kind of useless unless I wanted to create 10 hypotheticals for say question 18. I feel I must be missing something big here, can you guys offer any insight?

Best,
Barrett
 Claire Horan
PowerScore Staff
  • PowerScore Staff
  • Posts: 408
  • Joined: Apr 18, 2016
|
#42575
Hi Barrett,

Your diagram is looking good! I would also suggest having your chairperson slot clearly demarcated so you can make helpful inferences from the rule that provides that the chairperson must be from the group with exactly two representatives. That also allows you to make identify the outgroup slots as T or H once you know whether the chairperson is T or H.
 Leonard
  • Posts: 2
  • Joined: Jul 26, 2018
|
#48880
Hello i dont understand the reasoning behind the explanation that was given that if J or M was picked then Q R S has to be picked. What happened to F? Why cant it be FJM and then it can be Q and R or Q and s?
 Adam Tyson
PowerScore Staff
  • PowerScore Staff
  • Posts: 5153
  • Joined: Apr 14, 2011
|
#48939
Thanks for the question, Leonard, and thanks for joining us in the Forum! That inference about J and M was specifically when either of them is selected as the chairman. Remember that the group from which the chairman is selected can only have two members, so if one of them is the chairman then the two of them will make up the entire group.

The two groups you suggested with F, J & M would both be fine, so long as none of them was the chairman! That rule about the chairman can be easy to forget, so be sure to use the little subscript C in your diagram to help you remember it.

Keep up the good work, and keep coming back here as needed. We'll be here when you need us!

Get the most out of your LSAT Prep Plus subscription.

Analyze and track your performance with our Testing and Analytics Package.